Thanks for your reply.
Yes I have figured that out as well. Let me explain.
I know the rotation matrices in terms of the Pauli matrices, i.e R_x(\theta) = e^{-i\sigma_x /2} and the rotation matrices for \sigma_y and \sigma_z follows in the same manner. I could also prove that...
Hey,
(I have already asked the question at http://physics.stackexchange.com/questions/244586/bloch-sphere-interpretation-of-rotations, I am not sure this forum's etiquette allows that!)
I am trying to understand the following statement. "Suppose a single qubit has a state represented by the...